ash1374

New Member
ارسال ها
253
لایک ها
422
امتیاز
0
پاسخ : ماراتن جبر (سطح پیشرفته)

یه راه طولانی تر هم تغییر متغیر x+y=2a[SUP]2[/SUP] و ایده های هندسیه که یکم طولانیه ولی بامزه ست.

همه ی توابع
رو پیدا کنید که برای هر x,y حقیقی :
 

math

New Member
ارسال ها
1,129
لایک ها
1,096
امتیاز
0
پاسخ : ماراتن جبر (سطح پیشرفته)

یه راه طولانی تر هم تغییر متغیر x+y=2a[SUP]2[/SUP] و ایده های هندسیه که یکم طولانیه ولی بامزه ست.

همه ی توابع
رو پیدا کنید که برای هر x,y حقیقی :
من یه راه حل براش این جا گذاشتم :

http://www.artofproblemsolving.com/Forum/viewtopic.php?p=3391948#p3391948

نکته جالب اینه که این سوال انتخاب تیم ایران سال 2007 بوده و در سال 2005 در ml بیان شده بوده !!!!


http://www.artofproblemsolving.com/...ear=2007&sid=26cbd228531402be110acf1093d3babb


http://www.artofproblemsolving.com/Forum/viewtopic.php?f=38&t=38603
 

nima-1376

New Member
ارسال ها
63
لایک ها
53
امتیاز
0
پاسخ : ماراتن جبر (سطح پیشرفته)

یه راه طولانی تر هم تغییر متغیر x+y=2a[SUP]2[/SUP] و ایده های هندسیه که یکم طولانیه ولی بامزه ست.

همه ی توابع
رو پیدا کنید که برای هر x,y حقیقی :
راهنمایی
 

ash1374

New Member
ارسال ها
253
لایک ها
422
امتیاز
0
پاسخ : ماراتن جبر (سطح پیشرفته)

سوال بعدی:
چند جمله ای هایی با ضرایب صحیح هستند که
. ثابت کنید درجه ی یکی از این دو چندجمله ای 1 است.
 

mahdi math

New Member
ارسال ها
152
لایک ها
61
امتیاز
0
پاسخ : ماراتن جبر (سطح پیشرفته)

سوال بعدی:
چند جمله ای هایی با ضرایب صحیح هستند که
. ثابت کنید درجه ی یکی از این دو چندجمله ای 1 است.
خب جواب من اینه :
اولا اگه دو چند جمله ای معرفی شده در سوال رو به شکل زیر بنویسیم

آنگاه میدانیم اگر مقدار تابع g را در بزرکترین توان x در تابع f قرار ذهیم رگترین توان تابع fog بدست می آید که در ساده ترین شکل حضور دارد(توانی که حتما وجود دارد
حال بزرگترین توان تابع fog یا در واقع درجه ی آن برابر mn است )
در عبارت تابع ترکیبی فوق بزرگترین توان 1393 هست پس
mn=1393
یعنی 4 حالت وجود دارد
m=7 n=199
n=7 m=199
n=1 m=1393
n=1393 m=1
به ازای دو حالت دوم حکم اثبات میشود پس دو حالت اول را رد میکنیم
برای x با توان 1 در fog باید جمله درجه 1 در g در مکان جمله درجه 1 در f قرار گیرد
از طرفی چون در تابع fog جمله ای با توان 199 نداریم و تنها شکل آن جمله درجه 1 در g در مکان جمله درجه 199 در f است پس باید ضریب آن صفر شود اگر چنین حالتی پیش آید جمله ای با درجه 1 در fog نخواهیم داشت پس این حالت غیر قابل قبول است
به همین ترتیب حالت دیگر را رد میکنیم (این بار جمله درجه 199 با ضریب صفر بدست میاید که غ غ ق است و ردمیشود)
و بدین ترتیب حکم ثابت میشود
ببخشید در تعریف g و f جملات درجه صفرو لحاظ نکردم خودتون دقت کنید:15::224:
 
آخرین ویرایش توسط مدیر

ash1374

New Member
ارسال ها
253
لایک ها
422
امتیاز
0
پاسخ : ماراتن جبر (سطح پیشرفته)

خب جواب من اینه :
اولا اگه دو چند جمله ای معرفی شده در سوال رو به شکل زیر بنویسیم

آنگاه میدانیم اگر مقدار تابع g را در بزرکترین توان x در تابع f قرار ذهیم رگترین توان تابع fog بدست می آید که در ساده ترین شکل حضور دارد(توانی که حتما وجود دارد
حال بزرگترین توان تابع fog یا در واقع درجه ی آن برابر mn است )
در عبارت تابع ترکیبی فوق بزرگترین توان 1393 هست پس
mn=1393
یعنی 4 حالت وجود دارد
m=7 n=199
n=7 m=199
n=1 m=1393
n=1393 m=1
به ازای دو حالت دوم حکم اثبات میشود پس دو حالت اول را رد میکنیم
برای x با توان 1 در fog باید جمله درجه 1 در g در مکان جمله درجه 1 در f قرار گیرد
از طرفی چون در تابع fog جمله ای با توان 199 نداریم و تنها شکل آن جمله درجه 1 در g در مکان جمله درجه 199 در f است پس باید ضریب آن صفر شود اگر چنین حالتی پیش آید جمله ای با درجه 1 در fog نخواهیم داشت پس این حالت غیر قابل قبول است
به همین ترتیب حالت دیگر را رد میکنیم (این بار جمله درجه 199 با ضریب صفر بدست میاید که غ غ ق است و ردمیشود)
و بدین ترتیب حکم ثابت میشود
ببخشید در تعریف g و f جملات درجه صفرو لحاظ نکردم خودتون دقت کنید:15::224:
من که نفهمیدم منظورتون از "جمله درجه 1 در g در مکان جمله درجه 1 در f قرار گیرد" چیه؟؟؟

مثلا اگر f درجه 7 باشه اونوقت fog یه چیزی میشه مثل :
که همه ی جمله هاش میتونن جمله ی درجه 1 تولید کنن و مهم اینه که برایند این جملات چی بشه...
 

Mostafa_

New Member
ارسال ها
527
لایک ها
445
امتیاز
0
پاسخ : ماراتن جبر (سطح پیشرفته)

من یه راه حل براش این جا گذاشتم :

AoPS Forum - Nice functional equation • Art of Problem Solving

نکته جالب اینه که این سوال انتخاب تیم ایران سال 2007 بوده و در سال 2005 در ml بیان شده بوده !!!!


Iran Team Selection Test 2007 • Art of Problem Solving


AoPS Forum - f(x^2+y+f(y)) = f(x)^2 + 2y • Art of Problem Solving
یه سوال ml کجاست؟
 

AHZolfaghari

Well-Known Member
ارسال ها
935
لایک ها
1,654
امتیاز
93
پاسخ : ماراتن جبر (سطح پیشرفته)

همون mathlinks منظورشون هست .
یه نمونه دیگه :
سوال یک TST 2005 فرانسه همون سوال یک مرحله دو سال 1376 ایرانه !!
 
آخرین ویرایش توسط مدیر

mahdi math

New Member
ارسال ها
152
لایک ها
61
امتیاز
0
پاسخ : ماراتن جبر (سطح پیشرفته)

من که نفهمیدم منظورتون از "جمله درجه 1 در g در مکان جمله درجه 1 در f قرار گیرد" چیه؟؟؟

مثلا اگر f درجه 7 باشه اونوقت fog یه چیزی میشه مثل :
که همه ی جمله هاش میتونن جمله ی درجه 1 تولید کنن و مهم اینه که برایند این جملات چی بشه...
منظور جمله ایه که در f یا g توان x در اون یکه
 

ash1374

New Member
ارسال ها
253
لایک ها
422
امتیاز
0
پاسخ : ماراتن جبر (سطح پیشرفته)

منظور جمله ایه که در f یا g توان x در اون یکه
منظور از جمله درجه یک رو میدونم. اون جا که گفتین در مکان جمله ی درجه 1 f رو نمی فهمم. به بخش دوم پستم دقت کنین شاید مشکل حل بشه. دوباره اینجا می نویسمش:

"مثلا اگر f درجه 7 باشه اونوقت fog یه چیزی میشه مثل :
که همه ی جمله هاش میتونن جمله ی درجه 1 تولید کنن و مهم اینه که برایند این جملات چی بشه... "
 

math

New Member
ارسال ها
1,129
لایک ها
1,096
امتیاز
0
پاسخ : ماراتن جبر (سطح پیشرفته)

سوال بعدی:
چند جمله ای هایی با ضرایب صحیح هستند که
. ثابت کنید درجه ی یکی از این دو چندجمله ای 1 است.

راهنمایی :

از خواص چند جمله ایه مینیمال استفاده کنید .
 

Mostafa_

New Member
ارسال ها
527
لایک ها
445
امتیاز
0

ash1374

New Member
ارسال ها
253
لایک ها
422
امتیاز
0
پاسخ : ماراتن جبر (سطح پیشرفته)

راهنمایی :

از خواص چند جمله ایه مینیمال استفاده کنید .
تا حالا با مینیمال حلی برای این سوال ندیدم. میشه حل کامل بگذارید؟

به علاوه کلاً توی ماراتن ها بهتره حل کامل بگذارید و از لینک دادن و راهنمایی بپرهیزید(بجز کسی که سوال رو گذاشته که میتونه بعد از یک مدتی راهنمایی یا لینک بده).

---- دو نوشته به هم متصل شده است ----

اینی که گفتید یعنی چی؟
اگر a یک عدد حقیقی باشه f(x) با ضرایب گویا رو چند جمله ای مینیمال a میگن وقتی که بین همه ی چند جمله ای هایی که a ریشه ی اون هاست، f کمترین درجه را داشته باشد. این چند جمله ای خیلی خواص خوبی داره...
 

mohy1376

Well-Known Member
ارسال ها
418
لایک ها
311
امتیاز
63
پاسخ : ماراتن جبر (سطح پیشرفته)

یه سوال؟
با استفاده از چه نامساوی اینجوری شد؟
 

AHZolfaghari

Well-Known Member
ارسال ها
935
لایک ها
1,654
امتیاز
93
پاسخ : ماراتن جبر (سطح پیشرفته)

یه سوال؟
با استفاده از چه نامساوی اینجوری شد؟
باید مخرجو کوچک کنیم دیگه . رو هر کدوم از پرانتزهای زیر رادیکال مخرج حسابی هندسی بزن بعد رادیکال x رد از صورت و مخرج ساده کن. فکر کنم این جوریه
 

mohy1376

Well-Known Member
ارسال ها
418
لایک ها
311
امتیاز
63
پاسخ : ماراتن جبر (سطح پیشرفته)

باید مخرجو کوچک کنیم دیگه . رو هر کدوم از پرانتزهای زیر رادیکال مخرج حسابی هندسی بزن بعد رادیکال x رد از صورت و مخرج ساده کن. فکر کنم این جوریه
[FONT=Verdana, Arial, Helvetica, sans-serif]آره جواب داد.دیگه فک نکن ،مطمئن باش.[/FONT]:95:
 

ash1374

New Member
ارسال ها
253
لایک ها
422
امتیاز
0
پاسخ : ماراتن جبر (سطح پیشرفته)

سوال بعدی:
چند جمله ای هایی با ضرایب صحیح هستند که
. ثابت کنید درجه ی یکی از این دو چندجمله ای 1 است.
راهنمایی: مشتق بگیرید و از لم آیزنشتاین استفاده کنید.
 

mohy1376

Well-Known Member
ارسال ها
418
لایک ها
311
امتیاز
63
پاسخ : ماراتن جبر (سطح پیشرفته)

راهنمایی: مشتق بگیرید و از لم آیزنشتاین استفاده کنید.

خوب اگه مشتق بگیریم میشه:


طبق لم آیزنشتاین :

پس چند جمله ای
به حاصلضرب دو چند جمله ای با ضرایب صحیح تجزیه نمی شود از طرفی چون f,g صحیح الضرایب هستند و مشتق آنها هم ضرایب صحیح دارد پس یکی از 'f یا 'g باید عدد باشد پس یکی از f یا g درجه یک هستند.
 
آخرین ویرایش توسط مدیر

ash1374

New Member
ارسال ها
253
لایک ها
422
امتیاز
0
پاسخ : ماراتن جبر (سطح پیشرفته)

سوال بعدی(کمی ساده تر از قبلی):

اگر
ریشه حقیقی چندجمله ای
باشد ثابت کنید :
 

AHZolfaghari

Well-Known Member
ارسال ها
935
لایک ها
1,654
امتیاز
93
پاسخ : ماراتن جبر (سطح پیشرفته)

سوال بعدی(کمی ساده تر از قبلی):

اگر
ریشه حقیقی چندجمله ای
باشد ثابت کنید :
در واقع باید اثبات کنیم
از سه بزرگتر است و از چهار کوچکتراست.
برای قسمت دوم: داریم :
و همچنین :
پس
پس



که اینم بزرگ تر مساوی چهار هستش پس آلفا به توان شش از سه بزرگتر است پس براکت آلفا به توان شش میشه سه
 
بالا